Proving Boundary of Union is Subset of Union of Boundaries

  • Thread starter Thread starter teleport
  • Start date Start date
  • Tags Tags
    Union
Click For Summary
The discussion focuses on proving that the boundary of the union of two sets is a subset of the union of their boundaries, specifically showing that B(X ∪ Y) ⊆ B(X) ∪ B(Y). The initial approach involved using set algebra and the identity relating subsets and closures, but confusion arose regarding the meaning of closure versus complement. The participant clarified their understanding by recognizing that if a point x is in B(X ∪ Y), it must relate to points in both X and its complement. They ultimately resolved the issue by applying the relationship between complements of the sets, leading to a clearer understanding of the boundaries involved. The proof was successfully completed with this insight.
teleport
Messages
240
Reaction score
0

Homework Statement



I'm trying to prove that the boundary of the union of two sets is a subset of the union of the boundaries of the two sets. i.e. Show that B(X u Y) c= B(X) u B(Y).

Homework Equations





The Attempt at a Solution



Since I didn't know what to do here, I used the identity A c= B <==> cl(B) c= cl(A), and then a lot of set algebra to prove it. Given how simple the statement looks, I'm wondering if there is an easier way. Thanks.
 
Physics news on Phys.org
I don't see any need to deal with the closure. If x is a member of B(X \cup Y), what does that tell you about x? Use that to prove it must be a boundary point of X or a boundary point of Y.
 
Actually, didn't know what closure is, so confused the cl symbol I've seen, with complement. So, yea, when I wrote cl I meant complement.

O let me try this. If x in B(X u Y) then for all r>0 the r-neighbourhood of x contains at least some point p in X u Y and another point q in C(X u Y) , (C stands for complement). But now I'm having difficulty with the case where p in X but q in C(Y). This gives the possibility that x is in none of the boundaries of the sets. But if you say that x must be in the boundaries of the sets, then there is no need to solve the problem.
 
Im good, got it, just had to use C(X u Y) = C(X) n C(Y), so now it is clear what hapens to q. thank you.
 
Question: A clock's minute hand has length 4 and its hour hand has length 3. What is the distance between the tips at the moment when it is increasing most rapidly?(Putnam Exam Question) Answer: Making assumption that both the hands moves at constant angular velocities, the answer is ## \sqrt{7} .## But don't you think this assumption is somewhat doubtful and wrong?

Similar threads

  • · Replies 7 ·
Replies
7
Views
2K
  • · Replies 13 ·
Replies
13
Views
4K
  • · Replies 4 ·
Replies
4
Views
3K
  • · Replies 2 ·
Replies
2
Views
2K
  • · Replies 5 ·
Replies
5
Views
2K
  • · Replies 1 ·
Replies
1
Views
2K
  • · Replies 3 ·
Replies
3
Views
4K
  • · Replies 2 ·
Replies
2
Views
3K
  • · Replies 1 ·
Replies
1
Views
1K
  • · Replies 8 ·
Replies
8
Views
3K